¿Dónde están las funciones delta en Peskin & Schroeder eq. (11.67)?

En el modelo sigma lineal, el Lagrangiano viene dado por

(11.65) L = 1 2 i = 1 norte ( m ϕ i ) ( m ϕ i ) + 1 2 m 2 i = 1 norte ( ϕ i ) 2 λ 4 ( i = 1 norte ( ϕ i ) 2 ) 2 .

(por ejemplo, véase Peskin & Schroeder (P&S) página 349).

Al calcular perturbativamente la acción efectiva para este Lagrangiano, la derivada d 2 L d ϕ k ( X ) d ϕ yo ( X ) necesita ser computado. (por ejemplo, Ec. (11.67) en P&S):

(11.67) d 2 L d ϕ k ( X ) d ϕ yo ( X )   =   2 d k yo + m 2 d k yo λ [ ϕ i ϕ i d k yo + 2 ϕ k ϕ yo ] .

Mi pregunta: ¿dónde están dos funciones delta?

Si no entiende por qué hay que necesitarlos, escribo el cálculo completo:

d 2 L [ ϕ ] d ϕ a ( X ) d ϕ b ( y ) = d 2 d ϕ a ( X ) d ϕ b ( y ) { 1 2 i = 1 norte ( m z ϕ i ( z ) ) ( m z ϕ i ( z ) ) + . . . } = d d ϕ a ( X ) { i = 1 norte ( ( m z ϕ i ( z ) ) ( m z d d ϕ b ( y ) ϕ i ( z ) ) ) + . . . } = d d ϕ a ( X ) z { i = 1 norte ( ( m z ϕ i ( z ) ) ( m z d i b d ( z y ) ) ) + . . . } = d d ϕ a ( X ) { ( m z ϕ b ( z ) ) ( m z d ( z y ) ) + . . . } = ( m z d a b d ( X z ) ) ( m z d ( z y ) ) + . . . = d a b ( m z m z d ( X z ) ) d ( z y ) + . . . (1)

Puede ver dos funciones delta allí.

Puede ver dos funciones delta allí. A continuación afirmamos que X = y = z y tenemos

(2) d a b ( m z m z d ( 0 ) ) d ( 0 ) + . . .

Publicación relacionada de OP: physics.stackexchange.com/q/353669/2451

Respuestas (1)

  1. Primero, debemos centrarnos en lo que finalmente estamos tratando de calcular, es decir, la segunda derivada funcional de la acción (en oposición a, por ejemplo, la densidad de Lagrange)

    (A) d 2 S d ϕ α ( X ) d ϕ β ( y ) .
    Por lo tanto, el resultado (A) (que se resuelve en esta publicación de Phys.SE) contiene uno 4 Distribución delta de Dirac bidimensional (en lugar de, digamos, dos o cero). Para cumplir con la ec. (A), OP debe incluir una integración sobre z en su ec. (1).

  2. Es cierto que P&S no deja muy claro el punto anterior. De hecho, P&S está usando la notación de una derivada funcional del 'mismo espacio-tiempo' 1

    (B) d L ( X ) d ϕ α ( X )   :=   L ( X ) ϕ α ( X ) d m ( L ( X ) m ϕ α ( X ) ) + ,
    que no contiene distribuciones delta de Dirac, como se explica en mi respuesta a la publicación anterior de Phys.SE. Esto explica por qué no hay distribuciones delta de Dirac en la ecuación. (11.67).

--

1 Para completar, mencionemos que la mayoría de las fórmulas de P&S pueden explicarse mediante la notación de una derivada funcional (B) del 'mismo espacio-tiempo', pero la ec. (11.58) está incluso más allá de esta notación. Para dar sentido a la ec. (11.58) reemplaza todas las apariencias de L 1 en el derecho. con S 1 .

Escribí tu respuesta pero no entendí dónde están mis funciones delta δ(0).
La integración sobre z no es necesaria porque comencé con lagrangian en lugar de acción.
El Lagrangiano también contiene espacial z -integraciones. ¿Te refieres a la densidad lagrangiana?
Sí, me refiero a la densidad lagrangiana.
Bueno, debería ser la acción.